The total weight of a shipping crate is modeled by the function c = 24b + 30, * where c is the total weight of the crate with b boxes packed inside the crate. If each crate holds a maximum of 6 boxes, then what are the domain and range of the function for this situation?

Answers

Answer 1

The domain of the function is 0 ≤ b ≤ 6, and the range of the function is 30 ≤ c ≤ 174.

Understanding Domain of a Function

The function that models the total weight of a crate with b boxes inside is given as:

c = 24b + 30

We know that each crate can hold a maximum of 6 boxes. Therefore, the number of boxes inside the crate can only take values from 0 to 6.

Domain:

The number of boxes b can take values from 0 to 6. Therefore, the domain of the function is:

0 ≤ b ≤ 6

Range:

To find the range of the function, we need to consider the maximum and minimum values that c can take when

0 ≤ b ≤ 6.

When b = 0, the crate is empty, and the total weight of the crate is:

c = 24(0) + 30 = 30.

When b = 6, the crate is full with 6 boxes, and the total weight of the crate is:

c = 24(6) + 30 = 174.

Therefore, the range of the function is:

30 ≤ c ≤ 174

We can then say the domain of the function is 0 ≤ b ≤ 6, and the range of the function is 30 ≤ c ≤ 174.

Learn more about domain here:

https://brainly.com/question/26098895

#SPJ1


Related Questions

Ricky has 23 hours each week to dedicate to his classes. homework takes 6.5 hours and each class (c) is 1.5 hours long. how many classes does ricky take? which equation models the question? explain your thinking.



a) 23=6.5-1.5c b) 23=6.5+1.5c



c) 23=1.5+6.5c d) 23=1.5-6.5c

Answers

by dividing both sides by 1.5.

How many classes does Ricky take?

To solve the problem, we need to first determine the total amount of time Ricky spends in his classes. We know that each class is 1.5 hours long, so if he takes c classes, then he will spend a total of 1.5c hours on class time. In addition, we know that he spends 6.5 hours on homework. Therefore, the total amount of time Ricky spends on his classes and homework is:

Total time = Class time + Homework time

Total time = 1.5c + 6.5

We also know that Ricky has 23 hours per week to dedicate to his classes and homework. Therefore, we can set up the following equation:

Total time = 23

Substituting the expression for a total time from the first equation, we get:

1.5c + 6.5 = 23

Now we can solve for c:

1.5c = 23 - 6.5

1.5c = 16.5

c = 11

Therefore, Ricky takes 11 classes.

The equation that models the question is b) 23=6.5+1.5c. This equation correctly represents the total time Ricky spends on his classes and homework (23 hours), as well as the time he spends on homework (6.5 hours) and the time he spends in class (1.5c hours).

by dividing both sides by 1.5.

Learn more about Ricky taking classes.

brainly.com/question/31598666

#SPJ11

Suppose that in 1682, a man bought a diamond for $32. Suppose that the man had instead put the $32 in the bank at 3% interest compounded continuously. What would that $32 have been worth in 2003? In 2003, the $32 would have been worth $ (Do not round until the final answer. Then round to the nearest dollar as needed.)

Answers

If a man bought a diamond for $32 in 1682 and the man had instead put the $32 in the bank at 3% interest compounded continuously, then the value of the diamond in 2003 would be $554,311.

The given problem is related to exponential growth. In this problem, the continuous compounding formula will be used to find the value of $32 in 2003.

The formula for continuous compounding is given by:

A = Pert   Where,

P is the principal amount,

r is the annual interest rate,

e is the Euler's number which is approximately 2.71828, and

t is the time in years.

Using the formula, we get:

A = 32e^(0.03 x 321)

A = 32e^9.63

A = 32 x 17322.23

A = $ 554311.36

Thus, $32 invested at 3% compounded continuously from 1682 to 2003 would be worth $554,311.

To know more about interest, click on the link below:

https://brainly.com/question/30393144#

#SPJ11

In a survey, the planning value for the population proportion is p* = 0.25. How large a sample should be taken to provide a 95% confidence interval with a margin of error of 0.02? Round your answer up to the next whole number. How large a sample should be selected to provide a 95% confidence interval with a margin of error of 2? Assume that the population standard deviation is 30. Round your answer to next whole number.

Answers

The Sample size that is necessary for the selection is =7203

How to solve

Given that,

[tex]\hat p= 0.25[/tex]

[tex]1 - \hat p = 1 - 0.25 = 0.75[/tex]

margin of error = E = 0.01

At 95% confidence level the z is ,

\alpha = 1 - 95% = 1 - 0.95 = 0.05

[tex]\alpha / 2 = 0.05 / 2 = 0.025[/tex]

Z\alpha/2 = Z0.025 = 1.96 ( Using z table )

Sample size = n = [tex](Z\alpha/2 / E)2 * \hat p * (1 - \hat p)[/tex]

= (1.96 / 0.01)2 * 0.25 * 0.75

= 7203

Sample size =7203

In statistics, the sample size is the measure of the number of individual samples used in an experiment.

The size of the sample holds significant importance in any empirical study that aims to draw conclusions about a larger population based on a smaller sample.

Read more about sample size here:

https://brainly.com/question/30224379

#SPJ1

What is the product of the expression, 5x(x2)? (1 point)
25x2
10x
5x3
5x2

Answers

The product of the expression 5x(x²) is 5x³.


1. Write down the given expression: 5x(x²)
2. Apply the distributive property, which states that a(b + c) = ab + ac. In this case, we have a single term inside the parentheses, so the expression becomes: 5x * x²
3. Multiply the coefficients (numbers) together: 5 * 1 = 5
4. Multiply the variables together, which means adding the exponents since they have the same base (x): x¹* x² = x⁽¹⁺²⁾ = x³
5. Combine the result from steps 3 and 4: 5x³

The product of the expression 5x(x²) can be found by multiplying the coefficients (numbers) and adding the exponents of the variables (letters). In this case, we have 5 times x times x squared.

5 times x equals 5x, and x squared means x times x, so we can rewrite the expression as:

5x(x²) = 5x(x*x) = 5x³

So, the product of the expression 5x(x²) is 5x³.

Know more about expression here:

https://brainly.com/question/14083225

#SPJ11

A new sign is being designed for the cityâs skate park. Knowing the exact angles is necessary for fitting the sign where it will hang. The architect started to write in the angles, but went home sick before she could finish. It is up to you to fill in the missing angles. For 4 of the 8 missing angles, explain your answer

Answers

Using trigonometry, we can solve for the missing angles to find them as 18.43°, 45°, 45°, and 18.43°.

The sign is mounted on a sloped surface, which means that we'll need to use some trigonometry to find the missing angles.

Let's concentrate on the sign's upper right corner, where the letters x and y are absent from two perspectives. The magnitude of angle x may be determined using trigonometry.

Let's begin by sketching a right triangle that has an angle x. The triangle's two sides may be represented by the sign's vertical and horizontal lines, with the addition of a third side to join the top right corner of the sign to the sloping area below.

Since the sign is an octagon, we know that each interior angle is 135°. Therefore, the measure of angle y must be:

y = 180 - 135 = 45°

Now, let's look at the right triangle that includes angle x. We know that the hypotenuse of the triangle is the sloped surface of the sign, which has a length of 4.5 meters. We also know that the opposite side of the triangle is the height of the sign above the ground, which has a length of 1.5 meters.

Using trigonometry, we can find the measure of angle x by taking the inverse tangent of the opposite side over the adjacent side:

tan(x) = opposite/adjacent = 1.5/4.5 = 1/3

x = tan⁻¹(1/3) ≈ 18.43°

Therefore, the measure of angle x is approximately 18.43 degrees.

Hence, using trigonometry, we can solve for the missing angles to find them as 18.43°, 45°, 45°, and 18.43°.

To learn more about trigonometry, refer to:

https://brainly.in/question/2685053

#SPJ4

The width of the large size is 9.9 cm and its height is 19.8 cm.
The width of the small size bottle is 4.5 cm.
hcm
h =
4.5 cm
Calculate the height of the small bottle.
19.8 cm
9.9 cm
+
cm

Answers

Answer and Explanation:

The height of the small bottle can be calculated using the ratio of the width of the large and small bottles.

Ratio of width = Large bottle width / Small bottle width

Ratio of width = 9.9 cm / 4.5 cm

Ratio of width = 2.2

Therefore, the height of the small bottle can be calculated by multiplying the ratio of width with the height of the large bottle.

Height of small bottle = Ratio of width x Height of large bottle

Height of small bottle = 2.2 x 19.8 cm

Height of small bottle = 43.56 cm

Please help and explain if possibile

Answers

The missing lengths of triangles are 5in, 5mi, 13.9km,13.3mi respectively.

What is triangle?

A triangle is a closed, two-dimensional geometric figure with three straight sides and three angles.

What is Pythagorean theorem?

The Pythagorean Theorem is a fundamental theorem in Euclidean geometry that relates to the three sides of a right-angled triangle.

According to given information:

Using the Pythagorean theorem [tex](a^2 + b^2 = c^2)[/tex], we can solve for the missing side in each triangle.

Triangle 1:

[tex]a = 12 in\\\\c = 13 in\\\\a^2 + b^2 = c^2\\\\12^2 + b^2 = 13^2\\\\144 + b^2 = 169\\\\b^2 = 25\\\\b = \sqrt{(25)}\\\\b = 5 in[/tex]

Therefore, the length of the missing side in Triangle 1 is 5 in.

Triangle 2:

[tex]a = 4 mi\\\\b = 3 mi\\\\c = x\\\\a^2 + b^2 = c^2\\\\4^2 + 3^2 = x^2\\\\16 + 9 = x^2\\\\25 = x^2\\\\x = \sqrt{(25)}\\\\x = 5 mi[/tex]

Therefore, the length of the hypotenuse in Triangle 2 is 5 mi.

Triangle 3:

[tex]a = x\\\\b = 11.9 km\\\\c = 14.7 km\\\\a^2 + b^2 = c^2\\\\x^2 + 11.9^2 = 14.7^2\\\\x^2 = 14.7^2 - 11.9^2\\\\x^2 = 192.36\\\\x = \sqrt{(192.36)}\\\\x = 13.9 km[/tex]

Therefore, the length of the height in Triangle 3 is 13.9 km.

Triangle 4:

[tex]a = x\\\\b = 6.3 mi\\\\c = 15.4 mi\\\\a^2 + b^2 = c^2\\\\x^2 + 6.3^2 = 15.4^2\\\\x^2 = 15.4^2 - 6.3^2\\\\x^2 = 178.09\\\\x = \sqrt{(178.09)}\\\\x = 13.3 mi[/tex]

Therefore, the length of the height in Triangle 4 is 13.3 mi.

To learn more about triangle visit:

https://brainly.com/question/1058720

#SPJ1

After a windstorm, a leaning pole makes a 75° angle with the road surface. the pole casts a 15-foot shadow when the sun is at a 45° angle of elevation. about how long is the pole?

Answers

The pole is approximately 3.86 feet tall.

What is the length of a leaning pole that makes a 75° angle with the road surface, if it casts a 15-foot shadow when the sun is at a 45° angle of elevation?

Let's denote the height of the pole as "x" (in feet). From the problem, we know that the pole makes a 75° angle with the road surface, which means that the angle between the pole and the vertical is 90° - 75° = 15°.

Now, we can use the tangent function to find the height of the pole:

tan(15°) = x/15

Multiplying both sides by 15, we get:

x = 15 tan(15°) ≈ 3.86 feet

Learn more abut length

brainly.com/question/9842733

#SPJ11

20 pt if someone can answer this!!!
Pls help.

The median value is____

Answers

Answer:

The median value is 45.

Step-by-step explanation:

"The median is the middle number in a sorted, ascending or descending list of numbers"

The middle number here is 45

50

Step-by-step explanation:

I put the explanation on the attachment. please see it.

Sergey is solving 5x2 + 20x – 7 = 0. Which steps could he use to solve the quadratic equation by completing the square? Select three options

Answers

The quadratic equation where the squares had been completed is:

(x + 2)² = 27/5

How to complete squares?

Remember the perfect square trinomial:

(a + b)² = a² + 2ab + b²

now we have the quadratic equation:

5x² + 20x - 7 = 0

If we divide it all by 5, we will get.

x² + 4x - 7/5 = 0

Now we can rewrite this as:

(x² + 2*2*x )  - 7/5 = 0

Now we need to add 2² in both sides, we will get:

(x² + 2*2x + 2²) - 7/5 = 2²

(x + 2)² = 4 + 7/5

(x + 2)² = 27/5

There the square is completed.

Learn more about completing squares at:

https://brainly.com/question/10449635

#SPJ1

find the extremes of 4x−4y subject to condition x2 + 2y2 = 1

Answers

To find the extremes of 4x−4y subject to the condition x2 + 2y2 = 1, we can use the method of Lagrange multipliers.

First, we set up the Lagrange equation:

∇f(x,y) = λ∇g(x,y)

where f(x,y) = 4x-4y and g(x,y) = x2 + 2y2 - 1.

Taking partial derivatives, we have:

∂f/∂x = 4
∂f/∂y = -4
∂g/∂x = 2x
∂g/∂y = 4y

Setting these equal to their respective Lagrange multipliers, we have:

4 = 2λx
-4 = 4λy
x2 + 2y2 = 1

Solving for x and y in terms of λ, we get:

x = 2λ/4 = λ/2
y = -λ/4

Substituting these back into the constraint equation, we have:

(λ/2)2 + 2(-λ/4)2 = 1
λ2/4 + λ2/8 = 1
3λ2/8 = 1
λ2 = 8/3

Taking the positive and negative square roots of λ2, we have:

λ = ±2√2/3

Substituting these values back into x and y, we get:

For λ = 2√2/3:
x = (2√2/3)/2 = √2/3
y = -(2√2/3)/4 = -√2/6

For λ = -2√2/3:
x = (-2√2/3)/2 = -√2/3
y = -(-2√2/3)/4 = √2/6

Now we can find the extreme values of f(x,y) by plugging in these values of x and y:

f(√2/3, -√2/6) = 4(√2/3) - 4(-√2/6) = 4√2
f(-√2/3, √2/6) = 4(-√2/3) - 4(√2/6) = -4√2

Therefore, the maximum value of 4x-4y subject to the condition x2 + 2y2 = 1 is 4√2 and the minimum value is -4√2.

To learn more about derivative  visit;

brainly.com/question/30365299

#SPJ11

A portion of an electrical circuit is displayed next. the switches operate independently of each other, and the probability that each relay closes when the switch is thrown is displayed by the switch. what is the probability that current will flow from s to t when the switch is thrown

Answers

If you provide me with a specific circuit diagram and the relevant details, I would be happy to help you determine the probability of current flowing from s to t when the switch is thrown.

What is the probability of current flowing from s to t when the switch is thrown?

I apologize, but it seems that the circuit diagram you mentioned is not displayed here. Without the circuit diagram, it is not possible for me to provide a specific answer to your question.

However, in general, the probability of current flowing from s to t in an electrical circuit depends on several factors such as the voltage level, the resistance of the circuit components, and the state of the switches. If the switches are all closed, then the probability of current flowing from s to t will depend on the overall resistance of the circuit.

If you provide me with a specific circuit diagram and the relevant details, I would be happy to help you determine the probability of current flowing from s to t when the switch is thrown.

Learn more about specific circuit

brainly.com/question/30888688

#SPJ11

47 students are picking two activities to do over the weekend.
7 picked painting and sport.
6 did not pick painting or sport.
Twice as many students picked sport than painting as one of their activities.
Find the amount that picked sport and not painting.

Answers

Let's use the following variables:
- P = number of students who picked painting
- S = number of students who picked sport
- N = total number of students (47)

From the problem, we know that:
- 7 students picked both painting and sport, so P ∩ S = 7
- 6 students did not pick painting or sport, so (N - P - S) = 6, which simplifies to N = P + S + 6
- Twice as many students picked sport than painting as one of their activities, so S = 2P

We want to find the number of students who picked sport but not painting, which is S - 7 (since 7 students picked both). Substituting S = 2P into the equations above, we get:

N = P + 2P + 6
N = 3P + 6
47 = 3P + 6
41 = 3P
P = 13.67 (rounded to the nearest hundredth)

Since P must be a whole number, we can round up to P = 14. Then, S = 2P = 28, and S - 7 = 21. Therefore, 21 students picked sport but not painting.

3 Let a represent a positive number and let b represent a negative number. Tell whether each statement is True or False. A. The difference (a - b) could be negative. True False True False b. The difference (b - a) cannot be positive. C. The sum (a + b) could be positive. True False d. The sum (b + a) must be negative. True False​

Answers

Using various laws of integers we can say that if a is a positive integer and b is a negative integer, statement A is False, B is True, C s True and D is false.

Here we are given that a is a positive integer while b is a negative integer.

A. The statement says that the difference (a - b) could be negative.

According to the subtraction law of integers, when a negative number is subtracted from a positive number, that is we have

2 - (-3)

Here the 2 minus signs will make a positive to give

2 + 3 = 5

Hence (a - b) will be a positive number since b is negative.

B.

The difference (b - a) cannot be positive.

Since a is positive and b is negative, according to the above example we will get

-3 - 2 = -5

Hence it is true that the difference (b - a) can't be positive.

C.

The sum (a + b) could be positive.

Here, we can see that a is a positive number while b is a negative number. In the light of above example, we will get

2 - 3 = -1

Here the sum is nagative as 3 > 2, but if we had

3 + (-2), then the answer would have been 1. Hence (a + b) can be positive. Hence the statement is true.

D.

The sum (b + a) must be negative.

Integers have commutative properties. Hence a + b = b + a

Hence if a + b can be positive, then b + a can also be positive.

Hence the statement is False.

To learn more about Integer visit

https://brainly.com/question/1768254

#SPJ4

Find the solution and also verify your answer , under root 12 x 12 x - 4 is equals under root 4 x + 8

Answers

The solution to the given equation is x = -3/4 or x = 1.

What values of x satisfy the equation √(12x² - 4) = √(4x + 8)?

In order to find the solution, we start by squaring both sides of the equation to eliminate the square roots:

12x² - 4 = 4x + 8

Next, we simplify the equation by moving all the terms to one side:

12x² - 4x - 12 = 0

Now we can factor the quadratic equation:

4x² - x - 3 = 0

By factoring or using the quadratic formula, we find that the equation can be written as:

(4x + 3)(x - 1) = 0

Setting each factor equal to zero gives us the solutions:

4x + 3 = 0 or x - 1 = 0

Solving for x in each equation yields:

x = -3/4 or x = 1

Therefore, the solution to the given equation is x = -3/4 or x = 1.

Learn more about square roots

brainly.com/question/29286039

#SPJ11

In a regular tiling, if there are six polygons meeting at a vertex, then the angles at the vertex are _____ degrees

Answers

In a regular tiling, if there are six polygons meeting at a vertex, then the angles at the vertex are 120 degrees.

This is because each regular polygon has interior angles that are multiples of 180 degrees divided by the number of sides. For a regular hexagon, which has six sides, each interior angle measures 120 degrees. When six regular hexagons meet at a vertex in a regular tiling, the total angle sum at the vertex is 720 degrees (6 times 120 degrees).

Since the angles must be divided equally among the six hexagons, each angle at the vertex is 120 degrees.

Learn more about interior angles:

https://brainly.com/question/24966296

#SPJ11

Casho went shopping for a new pair of sneakers because of a sale. The price on the tag was $25, but Casho paid $22. 50 before tax. Find the percent discount

Answers

The percent discount on the sneakers is 10%

Casho paid $22.50 before tax, despite the item's $25 tag price. The discount is the difference between the original price and the sale price, which is $25 - $22.50 = $2.50.

The discount is the difference between the original price and the discounted price, expressed as a percentage of the original price.

To find the percent discount, we divide the discount by the original price and multiply by 100:

Percent discount = (discount / tag price) x 100

Percent discount = ($2.50 / $25) x 100

Percent discount = 0.1 x 100

Percent discount = 10%

Therefore, the percent discount on the sneakers is 10%

Learn more about Discount here

https://brainly.com/question/3541148

#SPJ4

1 pts How much bubble wrap is needed to cover a cylindrical vase that is 16 inches tall with a diameter of 6 inches?​

Answers

415 square inches of bubble wrap to cover the cylindrical vase that is 16 inches tall with a diameter of 6 inches.

To calculate how much bubble wrap is needed to cover the cylindrical vase, you will need to find the circumference and height of the vase.

First, calculate the circumference of the vase using the diameter of 6 inches:
Circumference = π x diameter
Circumference = 3.14 x 6
Circumference = 18.84 inches

Next, calculate the height of the vase which is given as 16 inches.

To find the surface area of the vase, you will need to multiply the circumference by the height and add the area of the circular bases. The formula for the surface area of a cylinder is:

Surface area = 2πr² + 2πrh
where r is the radius and h is the height.

Since the vase has circular bases, we can find the area of each base by using the formula:
Area of circle = πr²

Now, let's find the radius of the vase:
[tex]Radius = \frac{diameter}{2}[/tex]
[tex]Radius = \frac{6}{2}[/tex]
Radius = 3 inches

So, the area of each base is:

Area of base = π x (radius)²
Area of base = π x 3²
Area of base = 28.27 square inches

The total area of the two bases is 2 x 28.27 = 56.54 square inches.

Now, let's find the surface area of the cylinder:

Surface area = 2πr² + 2πrh
Surface area = 2 x π x 3² + 2 x π x 3 x 16
Surface area = 113.1 + 301.44
Surface area = 414.54 square inches

Therefore, you would need approximately 415 square inches of bubble wrap to cover the cylindrical vase that is 16 inches tall with a diameter of 6 inches.

To know more about "surface area of a cylinder" refer here:

https://brainly.com/question/28575608#

#SPJ11

7. A rectangular prism has a volume of 135ft^3. The width of the rectangular prism is (2x+10)ft. The height of the rectangular prism is 5 times it's width. Write a expression that gives the length of the rectangular prism in feet?



A. 4(x+5)/27 B. 27/4(x+5)



C. (2x^2+100)/27. D. 27/(2x^2+100)

Answers

The expression that gives the length of the rectangular prism in feet is option D: 27/(2x^2+100).

What is the expression that gives the length of the rectangular prism in feet?

The volume of a rectangular prism is given by the formula V = lwh, where l is the length, w is the width, and h is the height.

We are given that the volume of the rectangular prism is 135ft^3, and the width is (2x+10)ft. Also, the height is 5 times the width, so h = 5w.

Substituting these values in the formula for the volume, we get:

135 = l(2x+10)(5w)

Dividing both sides by (2x+10)(5w), we get:

l = 135 / (2x+10)(5w)

l = 135 / [10(x+5)w]

Now we can substitute h = 5w:

l = 135 / [10(x+5)h/5]

l = 135 / [2(x+5)h]

l = 135 / [2(x+5)(5w)]

l = 135 / [10(x+5)^2]

Simplifying the expression, we get:

l = 27 / (2(x+5)^2)

Therefore, the expression that gives the length of the rectangular prism in feet is option D: 27/(2x^2+100).

Learn more about rectangular prism in feet

brainly.com/question/30956442

#SPJ11

Please help ASAP!!!!! In a certain Spanish class of 30 students, 11 of them play basketball and 15 of them play baseball. There are 10 students who play both sports. What is the probability that a student chosen randomly from the class plays basketball or baseball? Answer


should be a fraction in simplest form

Answers

The probability that a student chosen randomly from the class plays basketball or baseball is 8/15

Total number of students in Spanish class = 30

Student who plays basketball (A) = 11

Student who plays baseball (B) = 15

Student who plays both sports (A and B) = 10

To find a student who plays basketball or baseball (A or B)

(A or B)  = A + B -  (A and B)

(A or B) = 11 +15 -10

(A or B) = 16

P(A or B) = No. of favorable outcome/ Total no. of outcomes

P(A or B) = 16/30

In simplest form = 8/15

To know more about probability click here :

https://brainly.com/question/30034780

#SPJ4

Question 1(Multiple Choice Worth 4 points)


A funnel is shaped like a cone and is 4. 5 inches high and has a diameter of 6 inches. What is the volume of the funnel? Use 3. 14 for pi. Round your answer to the nearest hundredth. 10. 60 in3

42. 39 in3

63. 61 in3

169. 64 in3

Answers

The volume of the funnel is approximately 42.39 in³. The correct answer is option 2.

To calculate the volume of the funnel, which is shaped like a cone, we need to use the formula for the volume of a cone: V = (1/3)πr²h.

Given:
Height (h) = 4.5 inches
Diameter = 6 inches
Radius (r) = Diameter / 2 = 6 / 2 = 3 inches
Pi (π) ≈ 3.14

Now, plug the values into the formula:

V = (1/3) × 3.14 × 3² × 4.5
V ≈ (1/3) × 3.14 × 9 × 4.5
V ≈ 3.14 × 3 × 4.5
V ≈ 42.39 in³

So, the volume of the funnel is approximately 42.39 in³. The correct answer is option 2.

To learn more about volume, refer below:

https://brainly.com/question/1578538

#SPJ11

If one line passes through the points (-3,8) & (1,9), and a perpendicular line passes through the point (-2,4), what is another point that would lie on the 2nd line. Select all that apply. ​

Answers

One point that would lie on the second line is (0,-4). Another  possible point on the 2nd line is (0, 12).

To find the equation of the first line, we can use the slope-intercept form:

y = mx + b

where m is the slope and b is the y-intercept. The slope of the line passing through (-3,8) and (1,9) can be found using the formula:

m = (y2 - y1) / (x2 - x1)

m = (9 - 8) / (1 - (-3))

m = 1/4

Using one of the points and the slope, we can find the y-intercept:

8 = (1/4)(-3) + b

b = 9

So the equation of the first line is:

y = (1/4)x + 9

To find the equation of the second line, we need to use the fact that it is perpendicular to the first line. The slopes of perpendicular lines are negative reciprocals, so the slope of the second line is:

m2 = -1/m1 = -1/(1/4) = -4

Using the point-slope form, we can write the equation of the second line:

y - 4 = -4(x + 2)

y - 4 = -4x - 8

y = -4x - 4

To find a point that lies on this line, we can plug in a value for x and solve for y. For example, if we let x = 0, then:

y = -4(0) - 4

y = -4

So the point (0,-4) lies on the second line.

Therefore, another point that would lie on the second line is (0,-4).

More on points: https://brainly.com/question/29395724

#SPJ11

Select all of the following that represent the part of the grid that is shaded.



A ten-by-ten grid has 7 columns shaded.



A.


70


100


B.


7


10


C.


70


10


D.


0. 07



E.


0. 7

Answers

A ten-by-ten grid has 7 columns shaded. All of the following that represent the part of the grid that is shaded are : The correct answer is (A) 70 and (B) 7.

The information given in the problem tells us that a ten-by-ten grid has 7 columns shaded. Since there are a total of 10 columns in the grid, this means that 7/10 of the columns are shaded.

To express this as a percentage, we can divide 7 by 10 and multiply by 100:

(7/10) x 100 = 70%

Therefore, 70 represents the percentage of columns that are shaded in the grid. Option (A) is correct.

Alternatively, we can express the same proportion as a decimal by dividing 7 by 10:

7/10 = 0.7

Therefore, 0.7 represents the proportion of columns that are shaded in the grid. Option (E) is incorrect because it shows 0.7 as a fraction instead of a decimal.

Option (B) is also correct because it correctly identifies the number of shaded columns as 7. Option (C) is incorrect because it includes both the percentage and the number of shaded columns, which is redundant. Option (D) is incorrect because it shows the proportion of shaded columns as a decimal with an extra zero.

To know more about grid, refer to the link below:

https://brainly.com/question/29774894#

#SPJ11

Roxie plans on purchasing a new desktop computer for $1250. Which loan description would result in the smallest monthly payment when she pays the loan back?



12 months at 6. 25% annual simple interest rate



18 months at 6. 75% annual simple interest rate



24 months at 6. 5% annual simple interest rate



30 months at 6. 00% annual simple interest rate

Answers

The loan with the smallest monthly payment is the 30-month loan at 6% annual simple interest rate, with a monthly payment of $45.83.

To determine the loan with the smallest monthly payment, we need to calculate the monthly payment for each loan option and compare them.

We can use the formula for monthly payment on a simple interest loan:

monthly payment = (principal + (principal * interest rate * time)) / total number of payments

where:

principal is the amount borrowed (in this case, $1250)interest rate is the annual simple interest rate divided by 12 to get the monthly ratetime is the length of the loan in months

We can compute the monthly payments for each loan choice using this formula:

1. 12 Monthly interest rate = 0.0625/12 = 0.00521, monthly payment = (1250 + (1250 * 0.00521 * 12)) / 12 = $107.35

2. 18 months at 6.75%: monthly interest rate = 0.0675/12 = 0.00563, monthly payment = (1250 + (1250 * 0.00563 * 18)) / 18 = $81.96

3. 24 months at 6.5%: monthly interest rate = 0.065/12 = 0.00542, monthly payment = (1250 + (1250 * 0.00542 * 24)) / 24 = $66.14

4. 30 months at 6%: monthly interest rate = 0.06/12 = 0.005, monthly payment = (1250 + (1250 * 0.005 * 30)) / 30 = $45.83

Based on these calculations, the loan with the smallest monthly payment is the 30-month loan at 6% annual simple interest rate, with a monthly payment of $45.83.

Read more on simple interest on:

brainly.com/question/24429956

#SPJ4

Pythagorean theorem
I need help with my math

Answers

The height of the flagpole is 26.0 feet.

What is height?

Height is the vertical distance between two points.

To calculate the height of the flagpole, we use the formula below.

Formula:

h = √(l²-d²)............... Equation 1

Where:

h = Height of the flagpolel = Length of the wired = Distance of the wire from the ground to the base of the pole

From the question,

Given:

l = 300 feetd = 15 feet

Substitute these values into equation 1

d = √(30²-15²)d = √675d = 26.0 feet

Learn more about height here: https://brainly.com/question/12446886

#SPJ1

do you believe your children will have a higher standard of living than you have? this question was asked of a national sample of american adults with children in time/cnn poll. sixty-three percent answered in the affirmatve, with a margin of error or plys or minus 3%. assume that the true percentage of all american adults who beleive their children with have a hgiehr standard of living is .60

Answers

True percentage of all American believes that their children have higher standard of living with confidence interval of 95% is between 60% and 66% .

CI is the confidence interval

Answered in the affirmative =  63%

p is the sample proportion =0.63

z is the critical value from the standard normal distribution at the desired confidence level

Using attached z-score table,

95% confidence level corresponds to z=1.96

n is the sample size

Use the margin of error ,

Calculate a confidence interval for percentage of American adults who believe their children will have a higher standard of living.

A margin of error of plus or minus 3% means ,

95% confident that the true percentage falls within 3% of the sample percentage.

Using the formula for a confidence interval for a population proportion,

CI = p ± z×√(p(1-p)/n)

Plugging in the values, we get,

⇒ CI = 0.63 ± 1.96√(0.63(1-0.63)/n)

Solving for n, we get,

n = (1.96/0.03)^2 × 0.63(1-0.63)

⇒ n = 994.87

Rounding up to the nearest whole number, sample size of at least 995.

⇒ CI = 0.63 ± 1.96√(0.63(1-0.63)/995)

⇒CI = 0.63 ± 0.02999

95% confidence interval for the true percentage is,

⇒CI = 0.63 ± 0.03

⇒CI = (0.60, 0.66)

Therefore, 95% confidence interval that between 60% and 66% of all American adults with children believe that their children will have a higher standard of living.

Learn more about confidence interval here

brainly.com/question/16409874

#SPJ4

900,000=x+y+z
79,750=0. 08x+0. 09y+0. 01z
2x=z

Answers

Answer:

since 2x = z

replace z with 2x

900000 = x+y+z

900000 = x+y+2x

900000 = 3x+y - eqn (1)

79750= 0.08x +0.09y+0.01z

79750 = 0.08x +0.09y+0.01(2x)

79750 = 0.08x+0.09y+0.02x

79750 = 0.10x +0.09y - eqn(2)

from eqn(1)

900000 = 3x + y

y = 900000-3x - eqn(3)

substitute eqn(3) in eqn(2)

79750 = 0.1x +0.09y

79750=0.1x + 0.09(900000-3x)

79750=0.1x+ 81000 - 0.27x

collect like terms

79750 -81000 = 0.1x-0.27x

-1250 = -0.17x

to find x divide both sides by -0.17

x = -1250/-0.17 ~= 7353

since 2x = z

2*7353 = 14706

in eqn(3)

y = 900000-3x

y= 900000-3(7353)

y = 900000-22059

y = 877941

x =7353,y= 877941,z=14706

Part A: Sydney made $18. 50 selling lemonade, by the cup, at her yard sale. She sold each cup for $0. 50 and received a $3 tip from a neighbor. Write an equation to represent this situation. (4 points)



Part B: Daria made a profit of $21. 00 selling lemonade. She sold her lemonade for $0. 75 per cup, received a tip of $3 from a neighbor, but also had to buy each plastic cup she used for $0. 10 per cup. Write an equation to represent this situation. (4 points)



Part C: Explain how the equations from Part A and Part B differ. (2 points)

Answers

Part A: The equation to represent this situation is: 0.50x + 3 = 18.50
Part B: The equation to represent this situation is: 0.75x + 3 - 0.10x = 21.00
Part C: The equations differ in the following ways:
1. Sydney's equation involves only the price per cup and the tip, while Daria's equation also considers the cost of the plastic cups.
2. The price per cup for Sydney and Daria are different.

Part A: The equation to represent this situation is:

18.50 = 0.50x + 3

Where x represents the number of cups of lemonade sold.

Part B: The equation to represent this situation is:

21.00 = 0.75x + 3 - 0.10x

Where x represents the number of cups of lemonade sold.

Part C: The equations from Part A and Part B differ in that Part B takes into account the cost of each plastic cup used to serve the lemonade, while Part A only considers the revenue from selling each cup of lemonade and the tip received. This means that the profit in Part B is calculated after deducting the cost of each plastic cup from the revenue earned, while the profit in Part A does not account for any costs incurred.

To know more about equation, refer to the link below:

https://brainly.com/question/24870633#

#SPJ11

Quadrilateral FGHJ was dilated with the origin as the center of dilation to create quadrilateral F' G′ H′ J′.



Which rule best represents the dilation that was applied to quadrilateral FGHJ to create quadrilateral F' G′ H′ J′?



A. (x, y) à (5/7x, 5/7y)


B. (x, y) à (1. 4x , 1. 4y)


C. (x, y) à (x + 1, y + 2)


D. (x, y) à (x - 2, y + 1)









Which rule best represents the dilation that was applied to quadrilateral FGHJ to create quadrilateral F' G′ H′ J′?

Answers

The rule that best represents the dilation that was applied to quadrilateral FGHJ to create quadrilateral F'G'H'J' is option B, which is (x, y) à (1.4x, 1.4y).

What is the dilation rule used to create quadrilateral F'G'H'J' from FGHJ?

A dilation is a transformation that changes the size of an object without changing its shape. It is performed by multiplying the coordinates of each point by a scale factor.

In this case, the center of dilation is the origin, which means that the coordinates of each point are multiplied by the same scale factor in both the x and y directions.

The scale factor can be found by comparing the corresponding side lengths of the two quadrilaterals. In this case, the scale factor is 1.4, which means that the lengths of the sides of F'G'H'J' are 1.4 times the lengths of the corresponding sides of FGHJ.

Learn more about dilation

brainly.com/question/13176891

#SPJ11

In the last 215 days, builders have completed 700 m2 of the alligator habitat that will eventually be 1,200 m2. How much longer will it take to complete the alligator habitat?

Answers

In the last 215 days, builders have completed 700 m2 of the alligator habitat that will eventually be 1,200 m2.
It will take approximately 153 days to complete the remaining part of the alligator habitat.

Determine how much longer it will take to complete the alligator habitat, first, we need to find the rate at which the builders are working.
Calculate the work rate
The builders have completed 700 m2 of the 1,200 m2 alligator habitat in 215 days.
Work rate = (completed work) / (number of days)
Work rate = 700 m2 / 215 days = 3.26 m2/day (approximately)
Calculate the remaining work
The total area of the alligator habitat is 1,200 m2, and 700 m2 has been completed.
Remaining work = Total area - Completed work
Remaining work = 1,200 m2 - 700 m2 = 500 m2
Calculate the time to complete the remaining work
Time to complete = (remaining work) / (work rate)
Time to complete = 500 m2 / 3.26 m2/day ≈ 153.37 days
It will take approximately 153 days to complete the remaining part of the alligator habitat.

Read more about alligator habitat.

https://brainly.com/question/15823089

#SPJ11

It will take approximately 394 more days to complete the alligator habitat.

We can start by finding the proportion of the habitat that has already been completed:

proportion completed = 700 m^2 / 1200 m^2 = 0.5833

This means that there is still 1 - 0.5833 = 0.4167 (or 41.67%) of the habitat left to complete.

Next, we can use a proportion to find out how long it will take to complete the remaining 41.67% of the habitat:

215 days / 0.5833 = x days / 0.4167

Solving for x, we get:

x = 215 days * 0.4167 / 0.5833 ≈ 153 days

Therefore, the total time it will take to complete the alligator habitat is approximately 215 + 153 = 368 days, or about 394 more days from the start.

For more questions like Alligator click the link below:

https://brainly.com/question/154098

#SPJ11

Other Questions
Find area bounded by y = in(x)/x, y = 0, and x = e. (Express numbers in exact form. Use symbolic notation and fractions where needed.) Which are potential sources of error in the experiment? Check all that apply.estimating temperature to the nearest tenth of a degreeestimating the mass of the sample to the nearest tenth of a gramestimating the thickness of the foam cupsthe position of the cups of sand and water under the heat lampthe brand of light bulb used for the heat lampthe air temperature outside the labanswer A,B,D someone help me with ScreenTime graph hw Analyze and annotate the graph for what it shows and make meaning Type one paragraph on the Graph and what it shows andProvide a one paragraph opinion on the topic For a total of two paragraphs Each paragraph must include 5-to-7 sentences Starplus Limited is well known for its focus on customer satisfaction and this is highlighted in the company profile. Their "About Us" page on their website tells a story of service and growth, all centred around their customers An eighth-grade student estimated that she needs $9,500 for tuition and fees for each year of college. She already has $5,000 in a savings account. The table shows the projected future value of the account in five years based on different monthly deposits. Initial Balance (dollars) $5,000 $5,000 $5,000 $5,000 Monthly Deposit (dollars) $100 $200 $300 $400 Account Value in Five Years (dollars) $11,000 $17,000 $23,000 $29,000 Problem The student wants to have enough money saved in four years to pay the tuition and fees for her first two years of college. Based on the table, what is the minimum amount she should deposit in the savings account every month? A $200 B $100 C $300 D $400 1. Simplify (Write each expression without using the absolute value symbol. )|120+x| if x Practice Task 2. To assess the relevance of the literary text (The Gift of Magi by O. Henry), write your comments on the following (use a separate sheet for youranswers)1. The purpose of the author2. The tone and the mood of the selection3. The theme4. The language used5. The lesson conveyed6. Readability or appeal of the text to the reader7. Relevance of the text to the society. Which answer correctly summarizes jackson's main point in sections 1 and 5? (5 points) a the congress and the president have too much power to ensure the liberty of the people. b as president, jackson wants to explore the limits of power within the office he's been elected to. c government is only a force for good as long as it protects people's rights and stays within established limits. d the office of the president does not have the same limits as congress and so has a greater responsibility to the people. Which statement best completes this diagram?CauseThe Confederateforces win the Battleof Bull Run.Effect?3 of 3 QUESTIONSThe Confederacy shifts its largest armies to western states.The Confederacy takes control of most of the border states.The Union realizes it will not be able to easily defeat the Confederacy.The Union gives up its plans to attack Virginia by sea. Trigonometry homework help Reddish-brown colordont need a magnifying glass to see grainsgritty when drysticks to my fingers when wetdoesnt smell like anythingdries quicklydoes not get foamy with vinegardamp soil made a ball but it fell apart quicklyWhat type of soil 3. (3 points) For ordinary differential equationX =1- x6with > 0, compute the update Ax= x(t+h) - x(t) using Euler's method the implicit Euler method the midpoint method. Why is everyone responsible to protect earth At what value(s) of x does cos x = 8x? X= (Use a comma to separate answers as needed. Type an integer or decimal rounded to two decimal places as needed.) Use Newton's method to obtain the third approximation, X2, of the positive fourth root of 4 by calculating the third approximation of the right 0 of f(x)= x - 4. Start with x0 = 1. The third approximation of the fourth root of 4 determined by calculating the third approximation of the right of f(x) = x - 4, starting with x0 = 1, is (Round to four decimal places.) A researcher asked 933 people what their favourite type of TV programme was: news, documentary, soap or sports. They could only choose one answer. As such, the researcher had the number of people who chose each category of programme. How should she analyse these data?a. T-test b. One-way analysis of variance c. Chi-square test d. Regression Select all the correct answers.If the measure of angle is which statements are true?sin (0) = -The measure of the reference angle is 30.cos (0) = 3The measure of the reference angle is 45.tan (0) = -3The measure of the reference angle is 60. A charitable group will be shipping packages to an area of the country that has been devastated by tornadoes. they plan to ship two kinds of packages: food and clothing. they have donations of good and clothing and money. the money will be used to pay the shipping costs.they collected $126. each package of food costs $18 to ship and each package of clothing costs $14 to ship.write an inequality to represent this situation. How to fix "the action cannot be completed because the file is open in another program"? what did ships transport on the middle passage Select the two sentences that tell how Daniel Webster changed Paul Jennings's life. He asked Jennings to write a book. W B) He helped Jennings become a free man. He told Jennings how to save the portrait. O He discovered a plot to free 77 slaves. He let Jennings fight in the Civil War. He gave Jennings a job to support himself.